click below
click below
Normal Size Small Size show me how
BSN 246
Concepts of nursing wk 2-15
| Question | Answer |
|---|---|
| The health care provider is educating a 65-year-old patient on a new diagnosis of primary open-angle glaucoma (POAG). Which statement by the patient shows an understanding of the discussion? | “I will need to follow my treatment plan to prevent damage to the optic nerve.” |
| The nurse is caring for a patient with a family history of age-related macular degeneration (AMD), a medical history of high blood pressure, and a social history including daily cigar smoking. The patient asks if there is any way to prevent development o | “Reduce or quit smoking cigars.” “Keep your blood pressure under control.” |
| A nurse is caring for a patient who has prolonged pupil dilation related to ophthalmic medication usage. The nurse will monitor the patient for which signs of acute PACG? | Nausea, vomiting, and acute eye pain An acute attack of PACG can result in excruciating pain in or around the eye that may be accompanied by nausea and vomiting. |
| A nurse is caring for a patient who reports loss of peripheral vision that developed over time. Which diagnostic tests would the nurse anticipate the patient to undergo? | Tonometry Gonioscopy |
| A nurse is counseling a patient about to undergo cataract surgery. The patient will have cataracts removed from both eyes, but just the poorest eye will be operated on initially. Which statement demonstrates that the patient understands the procedure? | “If a complication arises in the eye with poorest vision, I will still have some vision in the better eye.” |
| The patient with acute angle-closure glaucoma is preparing for surgery. The nurse notes that the patient consented for a lens removal procedure. How should the nurse proceed? | Contact the surgeon for revision of the consent form |
| During an education session for a patient with a chronic visual sensory perception problem, which information should the nurse ensure the patient knows? | Details of the treatment regimen Long-term effects of the disease process How to recognize emergency care needs That food preferences and nutritional habits can affect eyesight |
| A patient with a visual impairment is being admitted to the hospital after abdominal surgery. As the nurse prepares the room for the patient, which action must be taken? | Take note of the furniture placement in the room |
| The nurse is caring for a patient with age-related macular degeneration (AMD) and refers the patient to a psychologist. Which responses by the nurse are appropriate when the patient questions the reason for the referral? | “The permanent loss of central vision can be very difficult.” “This is done to discuss the need for additional support systems.” |
| The nurse is assessing a patient with Ménière’s disease. The patient presents with complaints of vertigo. What additional sign or symptom would the nurse suspect? | Significant inability to function |
| A patient has central and functional hearing loss. The nurse understands that which finding may have contributed to this disorder? | No physical reason for hearing loss can be identified |
| A patient calls the office and reports that his 78-year-old spouse has suddenly lost hearing in one ear. Which recommendation by the nurse is correct? | Bring his spouse in to be evaluated immediately |
| The student nurse is caring for a patient with an acute attack of Ménière’s disease. Which finding indicates a need for further teaching of the student nurse? | The patient is placed in the dayroom to watch a favorite action movie. |
| A nurse receives orders for a patient with an acute attack of Ménière’s disease. Which drug order should the nurse question? | ACE inhibitors |
| A nurse is teaching a group of new nurses about working with patients with profound hearing loss. Which statement made by a new nurse requires correction? | “I should avoid using hand movements.” |
| Which innate immune functions provide immediate protection from foreign antigens? | Phagocytosis of identified foreign cells External secretions neutralizing antigens Stomach acid-destroying bacteria Identifying malignant cells as foreign |
| What is the most accurate conclusion by the nurse about a patient’s immune system when assessing well-healing wounds that were contaminated during a car crash? | Innate immunity is being effective. |
| Which patient finding should the nurse report to the provider? | Monocytes 5.1 × 109/L |
| Which humoral immune events separate it from cell-mediated immunity? | Recognition of antigen non-self-markers B lymphocytes releasing specific antibodies Binding of antibodies to foreign antigens Chemotaxis from immune complex formation |
| Match the type of immunity with the source from which it is obtained. | Natural active Disease Artificial active Immunization Natural passive Breast milk Artificial passive Serum injection |
| In which order does humoral immunity react when stimulated by an invading foreign antigen? | B lymphocytes become sensitized to foreign antigen. Complement assists with antibody attachment to antigen. White cells destroy antigen-antibody immune complexes. Memory cells are synthesized for future antigen recognition. |
| The nurse will anticipate administering a prescribed antibiotic to a patient with which infection? | Staphylococcus aureus |
| Which actions will the nurse avoid to prevent indirect transmission of pathogens when caring for a patient with an infection? | Handling medical equipment Changing patient linens Removing food trays Disposing of tissues |
| Which statement by the patient on droplet precautions indicates understanding of how to prevent the spread of infection? | “I’ll cover my mouth whenever coughing or sneezing.” |
| Which clinical manifestations of sepsis differentiate it from a systemic infection? | Oliguria Hypotension Confusion Hypothermia Low platelets |
| Which patient assessment indicates the patient has a chronic infection? | Persistent fatigue |
| Match the situation with its definition. | Isolation Separates those with disease from those without disease Quarantine Restricts movement of those exposed to an infectious agent Protective isolation Prevents exposure of person with compromised immunity from others |
| Which action by the nurse contributed to a patient developing a health care–associated infection (HAI)? | Use of alcohol-based hand cleaner prior to inserting an indwelling urinary catheter |
| Which action will the nurse take when caring for a patient with drug-resistant Clostridium difficile (C-Diff) to prevent spreading the infection to other patients? | Teach the patient to wash hands after toileting. |
| The nurse will take which action when caring for the patient admitted with influenza? | Place in a single room on droplet precautions |
| Which action will the nurse take when implementing standard precautions? | Donning gloves prior to administering a patient intramuscular injection |
| Which collaborative intervention does the nurse determine to implement for the patient with an infection, pain, and fever? | Acetaminophen |
| Which intervention does the nurse determine addresses the collaborative goal of “eradication of the pathogenic agent”? | Oseltamivir prescribed for influenza |
| Which action does the nurse determine to take based on findings of a localized, purulent, infected leg lesion? | Prepare the patient for incision and drainage. |
| Which patient findings does the nurse determine to report to the provider because they indicate progression of the patient’s infection to sepsis? | Change in mental status to mild disorientation Heart rate 102 beats per minute Urine output to less than 30 mL/hour Cold, clammy skin with pale appearance |
| When assessing a patient with a respiratory infection, the nurse will anticipate which abnormal findings? | Lung crackles Wheezes Greenish sputum Heart rate 102 bpm Oral temperature 101°F |
| What conclusion can be drawn by the nurse when a patient shows no improvement in assessment baseline after two weeks of treatment for an infection? | Collaborative interventions were ineffective. |
| Which statement by a patient can the nurse determine indicates the need for further teaching about a prescribed anti-infective medication? | “I can miss a dose as long as I take it the next time the dose is due.” |
| The nurse determines to take which action based on the patient’s assessment finding of cloudy, foul-smelling urine? | Encourage increased fluid intake. |
| Which finding in the patient’s medical record indicates that the patient with human immunodeficiency virus (HIV) infection has developed acquired immunodeficiency syndrome (AIDS)? | Onset of wasting syndrome |
| The nurse is caring for an HIV+ patient whose laboratory reports indicate a normal CD4 T-cell count and no physical symptoms. What should the nurse infer from this finding? | The patient is in the incubation stage of HIV infection. |
| Which manifestation would the nurse expect to find when assessing the gastrointestinal tract of an HIV+ patient in the crisis stage? | Diarrhea |
| The nurse is caring for a patient who is HIV+ and has been admitted to the hospital for treatment of community-acquired pneumonia. Which intervention will prevent complications related to the patient’s HIV infection? | Administer antiretroviral drugs on time |
| The nurse is reviewing the laboratory test results for a patient who is HIV+. The results show no viral load. Which information should the nurse include in patient education? | “Use barrier contraception during sexual contact.” |
| A patient with a new diagnosis of human immunodeficiency virus (HIV) infection begins receiving antiretroviral therapy. Which finding indicates the patient is having complications related to the treatment regimen? | Weight loss |
| The school nurse is preparing to discuss HIV and AIDS with a group of teenagers. Which aspects of disease prevention would be best suited for this age group? | Modes of transmission Avoiding exposure to sexually transmitted diseases Not assuming that HIV affects only certain groups |
| A patient with HIV comes to the health care provider’s office for a check-up, and the nurse notices the patient has lost 10 pounds since the last visit. Which factors should the nurse assess next? | Oral cavity CD4 T-cell count Nutritional status Gastrointestinal (GI) function |
| Which intervention would be most appropriate for the nurse to include in the plan of care for a patient having side effects from the antiretroviral drugs? | Administer antiemetics |
| Which patient reports are consistent with manifestations of bladder cancer? | Visible blood in urine Urge to void 20 times per day |
| A patient presents with gross hematuria accompanied by urinary urgency and increased frequency. Which diagnostic test would be ordered first? | Urinalysis |
| A patient recently diagnosed with bladder cancer asks the nurse about the extent of the cancer. The nurse can tell the patient that which diagnostic test may be used to evaluate the cancer? | A computed tomography (CT) scan |
| Match the patient bladder cancer diagnosis with the appropriate collaborative care treatment for which the nurse would prepare the patient. | Stage II - 3.partial cystectomy Stage III - 2.radical cystectomy Stage IV - 1. chemotherapy |
| A 35-year-old woman is diagnosed with stage IV bladder cancer and indicates that she is planning to have children someday. The nurse would give the patient further information on which cancer treatment? | Radiation |
| A patient is diagnosed with stage 0 bladder cancer and refuses treatment for fear of hair loss associated with chemotherapy. What is the appropriate response by the nurse regarding side effects for treatment of early-stage bladder cancer? | “Intravesical chemotherapy is not associated with hair loss.” |
| A patient with carcinoma in situ of the bladder is treated with Bacille Calmette-Guérin (BCG). Which lab finding would cause the nurse to contact the primary health care provider? | White blood cell (WBC) count of 4/μL |
| A patient with stage IV bladder cancer undergoes a radical cystectomy with urinary diversion. The patient is drowsy, but arousable, complains of 7/10 pain in the suprapubic area, hr82rr14o297t99.8Fbp124/78 | Give intravenous pain medication. |
| The nurse is caring for a patient after a radical cystectomy in the postanesthesia care unit. The patient is obtunded, has no urine output in the urostomy bag, hr62bp92/78rr10o288 | Administer oxygen via nasal cannula. |
| Which conditions may put a patient at risk for developing a fungal urinary tract infection (UTI)? | AIDS diagnosis Renal failure Chronic bacterial infections |
| Which infections of the urinary tract may manifest systemically as symptoms of fever, chills, and flank pain? | Upper urinary tract infection (UTI) Acute pyelonephritis |
| Which urinary symptoms would indicate to the nurse that a urinary tract infection (UTI) has progressed to renal failure? | Oliguria Flank Pain |
| The nurse is caring for an older adult patient who complains of cloudy urine. Urinalysis indicates a bacteria count of 5 × 102 CFU/mL. The nurse understands that which treatment is most appropriate for this patient? | Oral nitrofurantoin therapy |
| A patient presents with a fever of 102° F, fatigue, nausea, and urinary urgency. On percussion during physical examination, the nurse notes costovertebral tenderness on the left side. Urinalysis indicates bacteriuria. | Administer intravenous (IV) ampicillin. |
| A patient presents with urinary pain and urgency but no systemic symptoms. Urinalysis indicates a bacteria count of 103 CFU/mL. The patient’s chart indicates no coexisting obstruction or health concern. | Ciprofloxacin is used to treat complicated UTIs and for patients with frequently recurring UTIs. |
| A patient presents with suprapubic pain, fever, and chills. The nurse notes heart rate of 112 beats/min, blood pressure of 146/88 mm Hg, and oxygen saturation of 100%. Which assessment findings would be most concerning? | Elevated white blood cell (WBC) count An elevated WBC count indicates infection and would be most concerning to the nurse. |
| A patient who is 5’5” and 180 lb is diagnosed with a urinary tract infection (UTI). The patient reports drinking 50 ounces of water a day and regular bowel movements and urination throughout the day. | Tell patient to increase water intake to 90 ounces per day. The patient should be taught to increase fluid intake to at least 90 ounces per day. |
| The nurse is caring for a patient being treated with ciprofloxacin for a catheter-associated urinary tract infection (UTI). Which assessment finding would be most concerning to the nurse? | Costovertebral angle (CVA) tenderness |
| Order the pathophysiological process of postrenal acute kidney injury. | Bilateral Ureteral obstruction. Kidney dilation. Increase in hydrostatic pressure. Decline in kidney function Tubular atrophy |
| The nurse can identify that which patients are at risk of developing AKI due to intrarenal causes? | Aminoglycosides MRI with contrast Lupus |
| The nurse knows that a patient with AKI who has been unable to produce urine for 12 hours would be in which stage of AKI? | Failure |
| A nurse is caring for a patient with AKI that has had 250mL/day urine output for a number of months and BUN as well as creatinine levels have remained elevated. Which outcome would the nurse expect for this patient based on their symptoms? | Onset of chronic kidney disease |
| A patient presents with low urine output and a history of low blood pressure. The nurse suspects that the patient’s acute kidney injury (AKI) is due to which kind of cause? | Prerenal |
| When the nurse began care for a patient with acute kidney injury (AKI), the nurse noted a patient urinary output of 3 L/day, but over the course of a few days this output lowered. The nurse suspects that the patient has begun the recovery phase of | Decreased Creatinine Increased GFR Decreased BUN |
| A nurse would expect to see which result upon taking the blood pressure (BP) of a patient with a known diagnosis of chronic kidney disease (CKD)? | 180/100 |
| A patient with chronic kidney disease (CKD) has had lowered glomerular filtration rate (GFR) levels for months. The most recent levels were recorded as 14 mL/min. What should the nurse expect to be the next recommendation for this patient? | Hemodialysis Hemodialysis is a treatment to remove excess fluid, wastes, and electrolytes in a patient with end-stage renal disease and a GFR of 14 mL/min. |
| What should the nurse include when educating a group of middle aged adults about CKD? | Monitor blood pressure on a regular basis Have fasting blood glucose levels drawn every year |
| Which manifestations would the nurse expect to find in a patient with chronic kidney disease (CKD) who has elevated blood urea nitrogen (BUN) levels? | Fatigue Fatigue results from toxin buildup such as blood urea nitrogen. Nausea/vomiting Nausea and vomiting can happen due to a buildup of toxins such as blood urea nitrogen. Impaired thought processes |
| What should the nurse assess next after discovering that a patient with CKD has a potassium level of 7.0 mEq/L? | Telemetry monitor |
| A patient with chronic kidney disease (CKD) who presents with a bone fracture may have which common complication of CKD? | Osteomalacia |
| Which menu items would the nurse remove from the meal tray of a patient with chronic kidney disease who is not currently receiving dialysis? | Bananas Bananas are a high-potassium food and should be avoided Hard boiled eggs Hard boiled eggs are high in protein, and the yolk contains cholesterol |
| A patient with CKD, who receives hemodialysis three times weekly, has a urine output of 200 mL for the day. The nurse calculates the recommended daily fluid intake for this patient to be mL of fluid. | 800 Urine output + 600 mL [insensible loss] = recommended fluid intake; 200 mL + 600 mL = 800 mL of fluid. |
| Which nutritional food group limitations would the nurse recommend to a patient with CKD and renal osteodystrophy in order to help manage the diagnosis? | Protein Phosphorus |
| What is the recommendation regarding the use of diuretics for a patient with acute kidney injury (AKI) during the oliguric phase? | Not recommended, may be harmful Diuretic therapy is not recommended during the oliguric phase and can cause additional harm |
| A patient with late stage chronic kidney disease (CKD) wants to know why active-form vitamin D has been prescribed. How would the nurse explain the use of this drug therapy to the patient? | "The kidney is no longer activating vitamin D, which will prevent calcium from being absorbed." With late stage CKD, the kidney no longer activates vitamin D. This prevents the absorption of calcium in the gut and can lead to mineral and bone disorders. |
| The nurse instructs a patient with CKD and mineral bone disorder to take which medications with food in order to maximize function? | Vitamin D Calcimimetics This medication helps with secondary hyperparathyroidism caused by the low blood calcium levels. Phosphate binders phosphate binding occurs in the bowel. The phosphate is then eliminated with the stool. |
| A patient who will soon begin dialysis requests more information about how dialysis will fit into their life. The patient would like to remain involved with work (full-time) and family activities, including day tri | "Continuous ambulatory peritoneal dialysis (CAPD) is the best option for you." CAPD is typically performed 4 times a day and takes about 30-45 minutes to complete but activities can continue while dialyzing. |
| Some patients may feel weak and have low blood pressure immediately following hemodialysis (HD). How would the nurse explain the mechanism that caused these symptoms to the patient? | Rapid fluid shifts can occur during HD as fluid is removed from the body.The rapid shift in fluid volume can cause hypotension, which leads to feelings of weakness and dizziness. |
| 93-year old patient with end-stage renal disease (ESRD) refuses dialysis. The patient’s family insists the patient cannot make that decision on their own. The nurse talks with the patient and determines that the patient is mentally stable | Explain to the family that the patient is competent. Treatment would be burdensome to the patient and is not what the patient desires. |
| Which issues would the nurse take into consideration when discussing dialysis versus kidney transplantation as management strategies for a patient with end-stage renal disease? | The physical ability to receive a kidney transplant. The patient’s mental competency to make the decision. The reasoning behind the patient’s decision regarding their treatment. |
| A patient undergoing hemodialysis will likely have an arteriovenous fistula or graft. Which specific safety precautions would the nurse take to care for these patients, in regards to their dialysis access? | Interventions to prevent clotting of the vascular access Meticulous aseptic technique to prevent infection of vascular access Peripheral venous access and venipuncture should be in the contralateral extremity Blood pressure on opposite side |
| A new graduate nurse is caring for an older adult patient who has acute kidney injury. Which statement made by the new graduate nurse would warrant further teaching? | “The patient does not have a fever, so further assessment is not required.” |
| During assessment, a patient with CKD states her or she is taking stinging nettle (herbal supplement) for hay fever. How can the nurse improve the nursing care for this patient? | Allow the patient to have input Discuss the treatment plan and therapeutic regimen Discuss alternatives for hay fever treatment that do not modulate fluid volumes or blood pressure Inform patient about obtaining health care provider consent for herbal |
| The nurse is caring for a patient with AKI who has an infection. Which order should the nurse question? | Antibiotic therapy |
| A patient with AKI presents to the emergency department with burns. What are the major nursing education concerns for this patient upon discharge? | Preventing infection Preventing fluid loss Preventing electrolyte imbalances |
| Which signs and symptoms should the nurse tell a patient with chronic kidney disease (CKD) to report immediately? | Increasing lethargy Shortness of breath Weight change from 145 lb to 150 lb Development of headaches or dizziness |
| A nurse is caring for a patient with chronic kidney disease (CKD) who is on a drug therapy regimen. Which patient statement should prompt the nurse to provide further patient education? | “I will take my calcium and iron supplements at 3 p.m. since it is between meals.” |
| Which cell type is involved in the initiation of head and neck cancer? | Squamous cells |
| Which symptom would the nurse most likely observe when assessing a patient for late-stage head or neck cancer? | Labored breathing |
| A patient who was recently diagnosed with neck cancer is experiencing severe abdominal cramping. The nurse anticipates the health care provider ordering which diagnostic study to determine whether the tumor has spread to other regions? | Computed tomography (CT) scan |
| A patient presents with a cancerous tumor in the neck, no evidence of disease in lymph nodes, and no evidence of distant metastases. The nurse would educate the patient and family about which therapy? | Larynx-sparing surgery |
| A patient undergoes a total laryngectomy with radical neck dissection for stage IV laryngeal cancer. What is the role of the nurse in the care of the patient immediately after surgery to ensure adequate hydration? | Administering intravenous fluids |
| A patient states, “I am tired of receiving chemotherapy, and I am feeling very depressed.” Which responses by the nurse are likely to be most supportive? | "Tell me more about how the chemotherapy is making you feel." "What do you feel you should do about continuing the chemotherapy?" "Tell me about what is making you tired of receiving the chemotherapy." |
| Which strategy is likely to be the most helpful in facilitating communication immediately after a patient’s laryngectomy? | Use of a communication board |
| A patient is being discharged from the hospital after undergoing radical neck surgery for stage III neck cancer. When planning this patient’s discharge, it is most important for the nurse to coordinate with which members of the health care team? | Surgeon Psychologist Speech therapist Registered dietician |
| Immediately after a laryngectomy, it is a priority to have which item in the patient’s room? | Suctioning equipment |
| What happens to the patient physiologically during an apneic period? | Blood carbon dioxide levels rise |
| If a patient’s sleep apnea is left untreated, which serious consequences may develop? | Death, Marital problems, Difficulty holding a job |
| A patient reports daytime sleepiness and indicates that he thinks he has sleep apnea. What is the initial assessment performed to evaluate this patient’s suspicion? | Sleep history review |
| Which statement by the patient demonstrates an understanding of treatment for obstructive sleep apnea? | “CPAP can improve my concentration.” |
| A patient is diagnosed with obstructive sleep apnea and provided with a CPAP mask and device to use. The patient returns a month later and reports no reduction in symptoms. Which question by the nurse would elicit the most appropriate add | “Are you experiencing problems with your mask?” |
| What is an appropriate response to a patient with a history of heart failure who asks why it is important to treat sleep apnea? | “Untreated sleep apnea can worsen heart failure.” |
| Which early manifestation of lung cancer would the nurse expect to be reported when completing the patient history? | A constant cough |
| A patient presents with signs and symptoms indicative of lung cancer, and the healthcare provider has already identified a mass in the lungs on a chest x-ray. The provider still wants a more definitive diagnosis. For which diagnostic test sh | Biopsy |
| Match the characteristics to the stage of non-small cell lung cancer. | Stage I 1 cm tumor only detected in the lung Stage II 2 cm tumor detected in the lung with some lymph node involvement Stage III Spread of tumor throughout the chest wall and lymph nodes Stage IV Metastasis to the bone |
| Match the appropriate lung cancer treatment to patient scenario. | stage IB and otherwise healthy. Surgical resection Stage IA cancer and hemophilia. Stereotactic radiotherapy significant pain associated with bone lesions. Radiation Therapy Patient with stage IV cancer who has had chemotherapy. Immunotherapy |
| A patient is diagnosed with small cell lung cancer with a tumor spread to the chest wall, pericardium, and regional lymph nodes. The patient is distraught and asks the nurse, “Can you explain my treatment options?” The nurse should offer patient edu | radiation chemotherapy |
| A patient with lung cancer states to the nurse: “I have a friend with lung cancer who underwent chemotherapy and was cured. Why am I not receiving chemotherapy as well?” What is the best response by the nurse? | "There are different types of lung cancer. Each type responds best to a specific treatment." |
| A nurse is caring for a patient with lung cancer who has been coughing severely and having trouble breathing. Which nursing intervention is appropriate to ensure that the patient achieves effective breathing patterns? | Raise the head of the bed to high fowler’s position |
| A patient with lung cancer is ordered oxygen therapy at home. The patient tells the nurse a son who is a smoker lives in the home. Which is the best response for the nurse to use when teaching the patient? | “Does he plan to smoke around you?” |
| A patient receives a preliminary diagnosis of lung cancer after a positive chest x-ray result. The patient is teary when talking with the nurse. Which statement by the nurse is appropriate? | “Are you dealing with any work or family situations right now during this waiting period?” |
| A family member of a patient asks the nurse how a person develops pneumonia. What is the best response by the nurse? | "Pneumonia develops when bacteria invade the lower airways." |
| A patient with pneumonia is ordered an arterial blood gas (ABG). A family member asks about the purpose of this test. Which response by the nurse is best? | “To determine the oxygenation status of the patient.” |
| Match each diagnostic study for pneumonia to its purpose. | Identifies lung infection Chest X-ray Determines the exact cause for infection Sputum Culture Provides oxygen, carbon dioxide, and pH levels in the blood Arterial Blood Gas Confirms leukocytosis Complete Blood Cell Count with Differential |
| A patient arrives at the emergency department reporting flu-like symptoms that have progressively worsened over the past two to three days. The nurse bases the plan of care on which likely cause of the patient’s symptoms? | Viral infection |
| A 22-year-old patient with cystic fibrosis is receiving a pneumonia vaccine. The patient asks the nurse why it is necessary to have the vaccine at such a young age. What is the best response by the nurse? | “Because of your health history, you should receive the vaccine even though you are not 65 years old.” |
| A patient reports severe flu-like symptoms that have worsened over the past 36 hours. The patient is coughing intensely and has an oral temperature of 101.7 °F. The patient questions why she has not received any antibiotics. Which response by the nurse | “Antibiotics treat bacterial infections. Your infection is viral in nature.” |
| A patient with pneumonia reports a loss of appetite and a weight loss of six pounds over the last week. Which interventions by the nurse are most appropriate for this patient? | Instruct the patient to eat small, frequent meals. Instruct the patient to choose their favorite foods at mealtime. |
| A patient calls the clinic and reports upper respiratory symptoms. Which statement is most appropriate by the nurse? | "When did your symptoms begin?" |
| A nurse is caring for a patient in the medical unit with altered consciousness. Which nursing interventions should be included in the plan of care to prevent this patient from getting pneumonia? | Raise the patient's head of bed to 30 degrees. Reposition patient every two hours while in bed. Perform hand hygiene before and after providing patient care. |
| A patient with Mycoplasma pneumonia is being discharged to the home. Which statements should the nurse include in the teaching plan for this patient? | "You should get a yearly influenza vaccination." "You need to drink six to ten glasses of clear liquids per day." "You must take your antibiotic until it is gone regardless of how well you are feeling." |
| Match the type of chest trauma to its corresponding clinical manifestations. Uncoordinated chest movement sever dypnea and areas of chest pain=Flail CHest | puncture chest wound, labored breathing, and mediastinal shift. Hemothorax dyspnea and decrease in one lung space at the apex. Spontaneous pneumothorax Hemoptysis, respiratory distress, and fluid in the pleural cavity. Blunt trauma |
| A patient presents with a heart rate of 105 beats per minute, a respiratory rate of 25 breaths per minute, and oxygen saturation of 90%. Additionally, the patient complains of chest pain. On assessment, the chest is intact and auscultation reveals | Spontaneous pneumothorax |
| A patient is diagnosed with a secondary spontaneous pneumothorax and asks the nurse “How could this have happened?” What is the best response by the nurse? | "Your chronic bronchitis resulted in the development of air-filled blisters in the lung which then ruptured." |
| Match Dyspnea, increased vital signs, and absent lung sounds in the right lower lobe Mechanical ventilation Chest tube draining a large amount of blood, tachycardia, and BP 101/62 IV fluid replacement | Paradoxical chest movement, on oxygen but having rapid decompensation Needle decompression Tracheal deviation and respiratory distress Chest tube placement |
| Which patient situations represent medical emergencies, requiring immediate intervention by the nurse? | Deviated trachea and increasing dyspnea Dyspnea and hemoptysis with a chest x-ray indicating fluid in the pleural cavity The patient is conscious, dyspneic, and splinting the asymmetrical chest movement |
| A patient with chronic obstructive pulmonary disease (COPD) reports to the clinic with tachycardia, dyspnea, and chest pain. A chest x-ray indicates air in the pleural cavity and diminished lung volume. The patient’s chart indicates this is the fourth | Partial pleurectomy |
| Which individuals are at risk for developing tuberculosis? | A prison guard A student born in South Africa A homeless IV drug user An individual with a low socioeconomic status |
| A patient presents with flu-like symptoms including fever, chills, chest pain, and a cough with blood-tinged sputum. Which classifications of tuberculosis would the nurse anticipate? | Pulmonary tuberculosis The respiratory symptoms indicate the disease is pulmonary in nature. Active tuberculosis disease The active symptoms the patient is experiencing indicate that the patient has tuberculosis disease. |
| The nurse notes a 5 mm of induration following a tuberculin skin test. For which patient would this be considered a positive test? | A patient who has been on high dose prednisone therapy for over a month to treat multiple sclerosis |
| A patient is receiving INH, rifampin, PZA, and ethambutol for an active TB infection. The results of the susceptibility testing of his sample reveal that the bacteria are resistant to INH and rifampin. The patient asks how this will change | “You will now receive bedaquiline and levofloxacin instead of what you were taking.” |
| A patient with tuberculosis presents with a high-grade fever, chills, and pleuritic chest pain and reports losing 10 pounds over the last month. The patient reports photophobia and an inability to maintain normal daily functioning due to the s | Admission to the hospital Hospitalization is recommended for patients who are severely ill and cannot care for themselves. |
| A patient receiving INH, rifampin, PZA, and ethambutol develops diminished visual acuity and exhibits symptoms of color blindness. The patient asks which medication is causing the side effects. What is the best response by the nurse? | “Ethambutol.” |
| A patient with tuberculosis presents with a productive cough, dyspnea, and fatigue. A chest x-ray indicates fluid in the pleural cavity. Which is an appropriate goal for this patient? | Patient will have normal pulmonary function |
| A patient with latent tuberculosis infection and HIV says, “I have stopped taking my medication since I am not sick.” What is an appropriate response by the nurse? | “Taking the medication is critical to prevent the latent infection from becoming an active infection.” |
| Match the appropriate nursing management to the patient scenario. A patient with anorexia, recent 12 lb weight loss, fever, and hemoptysis. Airborne infection isolation | Giving a bath to a patient with a positive tuberculin test. Wear a HEPA mask An injured homeless man being discharged to homeless shelter. Provide education about TB risk factors |
| Which patient is at the highest risk for developing chronic obstructive pulmonary disease (COPD)? | A smoker who works as a coal miner whose mother had COPD |
| Place the physical manifestations of chronic obstructive pulmonary disease in the order in which they appear in the disease process. | 1) Productive cough every few days for a month 2) Decreased endurance during morning walks 3) Difficulty breathing on awakening in the morning 4) Altered chest expansion 5) Bluish skin discoloration |
| The nurse is caring for a number of patients in the unit. Which patient most likely has chronic obstructive pulmonary disease (COPD)? | A 65-year-old smoker with 1:2 anterior-posterior (AP) chest diameter. |
| A patient with chronic obstructive pulmonary disease (COPD) reports a chronic productive cough and an unintended weight loss of 15 lb over the previous month. Which recommendations should be made to this patient to address the current symptoms? | High protein diet Increased oral fluid intake Use of diaphragmatic breathing |
| A patient arrives at the hospital with a history of long-term exposure to caustic fumes. Assessment reveals a forced expiratory volume in 1 second/forced vital capacity (FEV1/FVC) ratio of 65% and a functional oxygen saturation of 88%. | 1) provide supplement oxygen 2) teach diaphragmatic breathing 3) administer intravenous corticosteroids 4) obtain a sputum culture |
| A patient with chronic obstructive pulmonary disease (COPD) presents with orthopnea and dyspnea that interferes with activities of daily living. On assessment, the forced expiratory volume in one second (FEV1) is 58%. Which Med | Fluticasone/salmeterol |
| A patient diagnosed with chronic obstructive pulmonary disease (COPD) experiencing shortness of breath and a cough asks the nurse the best way to prevent exacerbations. Which recommendations should the nurse make? | Effective handwashing Effective handwashing can prevent secondary infections and prevent acute exacerbations. Preventative vaccinations Preventative vaccinations can decrease the risk for secondary infections and prevent acute exacerbations. |
| The nurse is preparing to discharge a patient with chronic obstructive pulmonary disease (COPD) who reports weight loss of 10 lb in the past month, fatigue, and dyspnea at rest. Which recommendations should the nurse include in discharge teaching? | Adequate sleep Adequate sleep is necessary to ensure the patient has energy and is able to perform activities of daily living. High-calorie diet A high-calorie, high-protein diet and will promote energy. Oxygen supplementation |
| The nurse is developing a teaching plan for a patient with newly diagnosed chronic obstructive pulmonary disease (COPD) who lives alone. The patient has a 30-pack-year history of cigarette smoking, walks 30 minutes twice per week, and is on a fixed incom | Assessment of readiness for smoking cessation |
| List the pathophysiologic mechanisms that occur in asthma in order. | Exposure to trigger Blood vessels dilate Bronchioles constrict Lung dysfunction |
| A patient with a history of asthma presents with wheezing and dyspnea and reports tightness of the chest. On assessment, the patient appears agitated and is fidgeting. The patient’s vital signs are: blood pressure 160/90 mm Hg, t | Hypoxemia |
| A patient is experiencing an acute asthma attack. The nurse understands that which common triggers may be the cause of this attack? | Allergic reaction to cats Intake of dairy products Exposure to air pollution |
| During a routine wellness visit, a patient reports asthma symptoms occurring approximately three times per week. The symptoms do not interfere with the patient’s sleep, and on assessment, forced expiratory volume in 1 second is within normal range. | Short-acting β2 agonist A short-acting β2-agonist would be given to a patient with a mild asthma exacerbation. |
| The nurse is caring for a patient with a history of asthma who reports severe dyspnea and the presence of chest tightness for 3 days that has escalated in the last few hours. The nurse notes, functional oxygen saturation of 87%, inspiratory and ex | Provide supplemental oxygen. The patient's symptoms indicate a moderate, acute asthma exacerbation. Supplemental oxygen is administered to treat the patient's hypoxemia. Administer intravenous (IV) corticosteroids. |
| A patient is being seen in the emergency department for an acute asthma attack. Initial forced expiratory volume in 1 second (FEV1) is 70%. After the initial dose of albuterol, FEV1 is 75%. Which provider order would the nurse carry out first? | Repeat albuterol dose |
| A patient presents with nasal flaring, intercostal retractions, and chest tightness. The nurse notes diminished breath sounds over both lungs. After administering albuterol and intravenous magnesium sulfate, which assessment finding indicate | Inspiratory and Expiratory wheezing |
| The nurse is caring for a patient with chest tightness and difficulty breathing. The nurse notes accessory muscle use, wheezes, and a peak flow of 50%. After oral corticosteroids and a short-acting β2-agonist have been administered, which assessment find | Diminished breath sounds Diminished breath sounds indicate impeded air flow in the lungs and would require further treatment. |
| A patient with asthma presents with complaints of dyspnea on exertion. The nurse notes hyperresonance on percussion. Which action should the nurse take? | Teach the patient the pursed-lip breathing technique. |
| Which variable best differentiates between type 1 and type 2 diabetes? | Level of endogenous insulin |
| A pregnant patient is worried that she may develop gestational diabetes after speaking with a friend who developed this condition during a recent pregnancy. Which factors should the nurse assess to determine whether the patient is | -Obesity -Advanced maternal Age -Family history of diabetes |
| A nurse is caring for a patient newly diagnosed with type 2 diabetes who wants to learn about the condition. The nurse should initiate teaching by identifying which body function as being impaired? | Glucose metabolism |
| A patient with type 1 diabetes presents to the emergency department with generalized weakness, mild disorientation, increased thirst, and a fruity breath odor. Which other manifestations should the nurse anticipate if diabetic ketoacidosis (DKA) is | Kussmaul’s respirations, restlessness, orthostatic hypotension |
| A patient with type 1 diabetes inadvertently administered a dose of insulin that was too high in the morning and now has a serum glucose level of 60 mg/dL. When managing this patient, the nurse should anticipate abnormal assessment findings related to | Brain |
| A patient with type 2 diabetes, who is admitted with pneumonia, is most at risk for developing which acute complication associated with diabetes? | Hyperosmolar hyperglycemic syndrome (HHS) |
| The nurse is caring for a patient with diabetes who is also receiving treatment for chronic obstructive pulmonary disease. The nurse should obtain capillary blood glucose levels based on documentation of which medication in the patient’s chart? | Prednisone |
| A patient reports a new onset of increased thirst, hunger, and frequent urination. Which action should the nurse take first? | Obtain a random plasma glucose level. |
| A nurse is preparing a patient with suspected diabetes for a fasting blood glucose test. The nurse should ask the patient about which factors? | recent acute illness history of alcohol use history of recent exercise |
| The nurse is providing patient education to a high school athlete with diabetes who was seen in the emergency department after a syncopal episode during a basketball game. Which teaching interventions are appropriate for the nurse to provide? | "Carry a source of quick-acting carbohydrate with you." "Obtain a capillary blood glucose level before you exercise." |
| Which subjective assessment finding indicates a need for further patient education about reducing the risk of developing complications from diabetes? | Reports walking barefoot only when it is warm outside |
| A nurse is providing dietary teaching to a patient with diabetes. Which patient action indicates a good understanding of dietary restrictions? | Using artificial sweeteners for beverages |
| A 16-year-old patient with diabetes reports to the nurse that morning blood glucose levels have been averaging around 200 mg/dL but are within an acceptable range the rest of the day. Which situation should the nurse suspect? | The patient is experiencing the dawn phenomenon. |
| A patient with newly diagnosed type 1 diabetes tells the nurse that he or she likes to go camping, but is now afraid to go because of insulin needs. Which information should the nurse provide to the patient? | Insulin can be stored at room temperature. |
| The nurse administers a dose of NPH insulin to a patient at 8:00 am. At which time should the nurse provide a snack or meal? | 12:00 pm |
| A patient with diabetes is admitted to the hospital for a colon resection as treatment for cancer. Which actions should the nurse include in the plan of care to manage the patient’s diabetes? | Administer insulin subcutaneously as prescribed. Provide foot care every shift and assess for ulcers. Assess the abdominal incision for signs of infection. |
| Which interventions should the nurse perform for a patient with diabetes who appears confused and responds to questions with irritation? | Give the patient a snack. Check the patient's blood glucose level. |
| The nurse is caring for a patient admitted to the hospital with no history of diabetes but who has a blood glucose level consistently greater than 200 mg/dL. Which action should the nurse take? | Review the patient’s current medication list. |
| The nurse is educating a patient with type 1 diabetes about disease self-management. Which statement indicates the patient understands the instructions given? | "I will record my blood glucose levels in a log book and bring these to my appointments." |
| The nurse is educating a patient about proper foot care. Which statement made by the patient indicates a need for further teaching? | "I will soak my feet in Epsom salts every night." |
| Which instruction should the nurse include when teaching a patient with type 1 diabetes about self-management? | "Keep your fasting blood glucose level below 125 mg/dL." |
| The nurse is caring for an older adult who sustained a closed head injury and notes that the patient has not urinated in the past four hours despite drinking 500 mL of fluid and receiving IV fluids at a rate of 150 mL/hr. | Syndrome of inappropriate secretion of antidiuretic hormone (SIADH) |
| The nurse is monitoring a patient recovering from the removal of a brain tumor. The nurse notes that the patient has an increase in urinary output to 450 mL of very light urine in the past hour. Which condition should the nurse suspect in this patient? | Diabetes insipidus (DI) |
| A nurse is assessing a patient with suspected hypopituitarism. Which findings would suggest that the patient may have a deficiency of adrenocorticotropic hormone (ACTH)? | Fasting hypoglycemia and headache |
| A patient is admitted to the intensive care unit because of low urine output, increased body weight, lethargy, and confusion. Serum sodium level is <120 mEq/L. What should the nurse do first? | Administer 3% sodium chloride intravenously at a slow rate |
| The nurse is preparing a patient with suspected diabetes insipidus (DI) for a water deprivation test on the following day. Which information should the nurse provide the patient? | “Your weight will be measured before the test.” “We will need to collect your urine in the morning.” “You will need to refrain from drinking anything once the test is started.” “Multiple specimens of your urine will be collected throughout the day.” |
| A nurse is managing a patient with diabetes insipidus (DI). Which order would the nurse question? | Administer 3% sodium chloride by intravenous (IV) infusion. |
| The nurse is caring for a patient with diabetes insipidus (DI) who is receiving desmopressin by subcutaneous injection. Which assessment components should the nurse monitor to determine the effectiveness of this treatment? | Vital signs Intake and output Level of consciousness |
| A patient reports having headaches and nausea and appears lethargic on the fifth day after removal of a pituitary tumor. Which is the best explanation of these clinical manifestations? | The patient may be experiencing syndrome of inappropriate secretion of antidiuretic hormone (SIADH) caused by manipulation of the pituitary gland. |
| A patient recovering from a pituitary surgery reports persistent facial pain and headache, especially around the eyes. What are the best actions by the nurse? | Keep the patient in bed. Elevate the head of the bed |
| A female patient with a history of pituitary surgery is speaking with the nurse. The patient becomes tearful when discussing how the surgery has permanently affected her life. Which factors should the nurse take into consideration when addres | After pituitary surgery, some people require lifelong hormone replacement therapy. The patient may have infertility resulting from the surgery and might benefit from a support group. |
| Which assessment finding made by the nurse may indicate the onset of hypoparathyroidism? | Positive Chvostek's sign A positive Chvostek's sign indicates hypocalcemia, which occurs with hypoparathyroidism. |
| What should the nurse include in the assessment of a patient who presents with thyroid storm? | Heart rate Temperature Blood pressure |
| Which information would the nurse expect to see in the medical record of a patient with Graves’ disease? | Insomnia Exophthalmos History of systemic lupus erythematosus |
| A patient with a diagnosis of metastatic thyroid cancer is preparing for treatment. Which would be the best collaborative care treatment modality for the health care provider to prescribe for this patient? | Targeted therapy |
| A patient with hyperparathyroidism reports that she has started a vegan diet. What should the nurse include when teaching this patient about dietary management of the disease? | Include soybeans and tofu in your diet calcium can come from soybeans and tofu. Take a daily calcium supplement The patient should be instructed to take a daily calcium supplement to maintain bone strength. |
| The nurse is teaching a patient about management of hyperparathyroidism. Which statement made by the patient indicates a need for further teaching? | "It will be nice to not take any calcium pills anymore." The patient will need to take calcium supplements because osteoporosis may develop because calcium is pulled from the bones as a result of increased parathyroid hormone levels. |
| Which electrolyte should the nurse monitor when caring for a patient with hypoparathyroidism? | Serum calcium level The serum calcium level should be monitored at least three to four times per year in a patient with hypoparathyroidism. |
| A patient with thyroid cancer who underwent a total thyroidectomy is being discharged. What should the nurse include in the discharge instructions? | “Return to the clinic in one month to have thyroid hormone levels measured.” “Notify the health care provider of any purulent drainage from the incision.” |
| The nurse is assessing a patient who underwent a parathyroidectomy related to hyperparathyroidism and finds positive Chvostek’s and Trousseau’s signs. Which action should the nurse take next? | Check patient's serum calcium levels Positive Chvostek's and Trousseau's signs occur when the patient has hypocalcemia. Therefore the nurse will check the serum calcium level next. |
| A patient admitted to the hospital with thyroiditis asks the nurse why the health care provider is testing for systemic lupus erythematosus (SLE). What is the nurse’s best response? | "Patients with thyroiditis caused by a defect in the immune system often have other autoimmune diseases." Because thyroiditis has autoimmune origins, the patient may have other autoimmune diseases, such as SLE. |
| A nurse is caring for a patient with Cushing’s syndrome whose blood pressure is 160/100 mm Hg on two occasions. Which adrenal hormone is likely altered in this patient? | Mineralocorticoid excess may cause hypertension secondary to fluid retention and is the hormone that is likely altered in this patient with Cushing's syndrome. |
| Which is the most common cause of primary adrenal insufficiency? | Primary adrenal insufficiency, also known as Addison's disease, is caused by decreased production of corticosteroids related to hypofunctioning of the adrenal cortex. |
| Which is the most common cause of Cushing’s syndrome? | Long-term corticosteroid use |
| The nurse is caring for a patient with Addison’s disease who has abdominal pain, vomiting, and the following vital signs: temperature 101.1° F, heart rate 110, respiratory rate 22, and blood pressure 84/58. Which action should the nurse take first? | 2) Administer a 1000-mL IV bolus of normal saline solution - A patient with history of Addison's disease who has signs and symptoms of Addisonian crisis needs aggressive management with large volumes of isotonic fluid to reverse hypotension. |
| The nurse notes a “moon” face, abdominal striae, truncal obesity, and hirsutism during the assessment of a patient with Addison’s disease who takes daily oral corticosteroids. Which order by the health care provider would the nurse anticipate? | 1) Obtain a plasma adrenocorticotropic hormone (ACTH) level - High or normal ACTH levels indicate Cushing's syndrome, which would be helpful in confirming a diagnosis of Cushing's syndrome in this patient with Addison's disease, |
| The health care provider has ordered methylprednisolone, 75 mg, to be administered intravenously for a patient in Addisonian crisis. Methylprednisolone is supplied at 120 mg/2mL. How many milliliters would the nurse administer? | 1.3 |
| The nurse is caring for a patient with Addison’s disease who has abdominal pain, a craving for salt, and the following vital signs: temperature 101.3° F, heart rate 118, respiratory rate 24, and blood pressure 86/52. | Wash hands and maintain a calm environment |
| The nurse is providing discharge teaching to a patient with Cushing’s syndrome. Which statement made by the patient indicates the need for further teaching? | 4) "I should stop taking my medications because they caused Cushing's syndrome." even though corticosteroids cause Cushing's syndrome, the patient should not abruptly stop taking this medication as it could cause Addisonian crisis. |
| A patient with suspected Addison’s disease arrives in the emergency department, lethargic and confused. Which action by the nurse is the priority? | 1) Check the patient's medical alert bracelet - Because the patient may have Addison's disease, assessing the patient's medical alert bracelet will provide the best information about how to care for this patient. |
| Which criterion is necessary for a patient to progress from elevated blood pressure (BP) or prehypertension to a diagnosis of hypertension (HTN)? | A persistent BP elevation on at least two separate occasions |
| A patient with an adrenal tumor has consistent blood pressure readings of 152/86. Which one of the classifications would this disorder represent? | Secondary hypertension |
| Which snack selection increases the risk for primary hypertension? | French fries |
| Which diagnostic tests would be ordered to screen for kidney involvement in a patient with hypertension? | Urinalysis Creatinine levels Blood urea nitrogen levels |
| Which patient has the highest risk of hypertensive problems? | A 50-year-old obese male who smokes. |
| The nurse is caring for a patient with a blood pressure of 168/72. The patient complains of a headache and blurred vision. Which additional subjective information would indicate the presence of severe hypertension? | Angina Fatigue Dyspnea Dizziness Palpitations |
| A 65-year-old male presents with hypertension and reports that he works 6 days per week and cooks most food in a deep fryer for convenience. He swims 4 days weekly and sleeps 4 hours nightly. When discussing necessary lifestyle changes, whi | decrease work to 5 days or only 40 hours a week include a lightweight training regimen with swimming |
| A 55-year-old male presents with a 170/48 BP. He tells the nurse that he is currently going through a divorce. He admits he has been depressed and angry because of financial problems stemming from his divorce. | “Sometimes stress can cause an increase in blood pressure.” |
| The nurse is providing patient teaching to a 46-year-old with hypertension. Which patient meal selection indicates that the teaching was effective? | Grilled chicken with mushrooms and asparagus, baked potato, and skim milk. |
| A patient with hypertension has been prescribed an ACE inhibitor. Which information should the nurse include in patient teaching? | 1) you should avoid bananas and salt substitutes 3) notify the health care provider if you develop a persistent cough |
| A patient receiving loop diuretic drug therapy for hypertension reports dizziness when changing positions from sitting to standing. Which action should the nurse take? | Obtain orthostatic vital signs. Instruct the patient to switch to salt substitute. Tell the patient to stand slowly and with assistance. Encourage the patient to report symptoms to the health care provider. |
| Which information should the nurse include in the patient instructions related to antihypertensive drug therapy? | patient should take the medication at regular time intervals as ordered the pt may experience sexual problems as a side effect of the medication pts should be cautions when rising from a sitting to a standing position too rapidly |
| A patient with hypertension reports taking antihypertensive medications every other day. Which action should the nurse take first? | Verify the patient’s prescription |
| A new graduate nurse is obtaining a blood pressure reading on an adult patient. Which of the following actions by the new graduate nurse would require intervention? | Using a pediatric blood pressure cuff to take a blood pressure |
| A patient, who reports feeling light-headed after standing up, asks to use the restroom. Which action should the nurse take? | Provide assistance when getting out of bed Have patient sit up slowly and dangle legs on the side of bed before standing |
| The nurse is educating a patient with hypertension about symptoms to report after discharge. Which statement made by the patient indicates teaching was effective? | "I will call my health care provider if I have any headaches or blurred vision." |
| A patient, who is newly diagnosed with hypertension, expresses concern about the condition and relates family history of strokes. Which information should the nurse provide to this patient? | Home blood pressure checks are important. Risk for stroke decreases with healthy lifestyle. Hypertension cannot be cured but can be controlled. |
| A patient, who was recently prescribed with a new antihypertensive medication, reports having frequent syncopal episodes. Which question should the nurse ask the patient first? | "How often do you take your medication?" Rationale: Knowing the frequency of dosing will help the nurse determine the cause of the patient's symptoms. |
| How would the nurse explain to the patient with coronary artery disease (CAD) why chronically elevated levels of C-reactive protein (CRP) are a good predictor of CAD? | “Chronically elevated CRP levels are associated with plaque formation.” |
| The nurse identifies which patient as being at greatest risk for coronary artery disease (CAD)? | 80-year-old woman with hyperlipidemia |
| When assessing a patient with coronary artery disease (CAD), which finding is expected? | Swelling in the legs |
| A patient returns to the office after starting statin therapy. Which statement by the patient would indicate the need for further assessment by the nurse? | “I am having trouble walking.” |
| A patient at risk for coronary artery disease (CAD) has a serum triglyceride level of 160 mg/dL and a low-density lipoprotein (LDL) level of 180 mg/dL. Which dietary recommendations by the nurse are crucial for reducing the risk for CAD in this patient? | Eliminate alcohol Eat more broccoli Eat more shellfish |
| The nurse is assessing a patient with coronary artery disease (CAD) for response to lipid-lowering therapy. Which question is a priority for the nurse to ask in order to determine the effectiveness of the therapy? | “Have you had any chest pain in the last week?” |
| A patient at high risk for development of coronary artery disease (CAD) indicates that the necessary lifestyle changes to decrease the risk for disease are too difficult. How should the nurse respond to help the patient understand the importance of m | “Are you aware of the major risk factors for CAD that you have?” “What aspect of your current lifestyle is most important for you to maintain?” |
| Which statement by the patient with coronary artery disease (CAD) demonstrates understanding of how to modify dietary fat intake by substituting monounsaturated or polyunsaturated fats for saturated fats? | “I ordered a cheese with egg white sandwich.” |
| A patient’s spouse asks the nurse about alternative methods to reduce coronary artery disease (CAD) risk factors because of potential side effects of a medication prescribed for her husband. Which response by the nurse is appropriate? | Discuss gradually increasing physical activity. |
| Which event occurs when myocardial oxygen demand exceeds oxygen supply? | Lactic acid is formed and irritates myocardial nerve fibers. |
| Chronic stable angina is manifested by which common signs and symptoms? | redictable onset Pain lasting 10 minutes Decreasing discomfort with rest |
| Which factors increase myocardial oxygen demand? | Anemia Heart rate Cocaine use Increased body temperature |
| Which finding would cause the nurse to alert the health care provider before a patient undergoes percutaneous coronary angiography? | The patient has preexisting renal condition. |
| The nurse is caring for several patients with a history of chronic stable angina. After receiving the previous shift’s report, which patient should the nurse assess first? | Patient returning from cardiac catheterization |
| A 67-year-old man with chronic stable angina is taking sublingual nitroglycerin. Which symptom would help the nurse assess the effectiveness of this medication? | Decreased blood pressure |
| The nurse is assessing a female patient who presents with complaints of fatigue and increasing episodes of angina. Which questions should the nurse include in the assessment? | "When was your last menstrual period?" - Once a woman reaches menopause, microvascular angina is more common, and chest pain is related to myocardial ischemia |
| A patient with diabetes is trying to reduce risk factors for chronic stable angina. Which is the best action for the nurse to recommend? | Gradually increase physical activity. |
| A patient has had type 2 diabetes for the last 22 years and is beginning to have episodes of chronic stable angina. Which statements by the nurse are appropriate for patient education? | “Your physician may order an exercise stress test.” “Diabetes increases the risk for chronic stable angina.” “You will need to have an electrocardiogram (ECG) to determine your current cardiovascular status.” |
| Which statement describes how unstable angina differs from chronic stable angina? | Unstable angina has a random onset. |
| Unstable angina is considered an emergency because of which pathophysiological occurrence? | Platelet aggregation blocks the artery. Unstable angina results from a ruptured plaque that causes platelet aggregation, which forms a thrombus. |
| A patient has been diagnosed with acute coronary syndrome. Which assessment findings would the nurse expect to see? | Pain occurs at rest. Pain may radiate to other areas. Severe, immobilizing chest pain. |
| A male patient diagnosed with acute ST-segment-elevation myocardial infarction (STEMI) is receiving thrombolytic therapy. Which assessment finding by the nurse would require immediate intervention? | The skin is pale and cool with bilateral pedal pulses faintly palpable. |
| Which patients are more likely to require off-pump coronary artery bypass (OPCAB) surgery instead of coronary artery bypass graft (CABG) surgery? | Patient with renal complications whose urine output is 50 mL per day Patient with left ventricular heart failure, obesity, and poor wound healing Patient with history of an ischemic stroke who has difficulty using the right arm |
| A patient comes to the emergency department with complaints of chest pain. The initial ECG shows ST-segment elevation in more than three leads. Which intervention should the nurse perform first? | Administer oxygen and nitrates. |
| A patient is being discharged with prescriptions for aspirin and clopidogrel after percutaneous coronary intervention (PCI) to treat acute coronary syndrome. The patient wants to know why these medications need to be taken if the artery is open a | Aspirin and clopidogrel can prevent another clot from forming. |
| A patient is complaining of angina that has increased in intensity. The patient’s vital signs are as follows: blood pressure 94/52, heart rate 122, respiratory rate 20, temperature 99.9, and oxygenation saturation 93%. An | Notification of the health care provider |
| A patient comes to the emergency department with complaints of chest pain, and heart monitoring is initiated. The nurse notes T-wave inversion on the electrocardiogram (ECG). The patient questions the nurse about what this means. | "You are having decreased blood flow to part of the heart." |
| The Emergency Medical Service (EMS) was called to assess a person who had collapsed. The patient’s electrocardiogram (ECG) has been transmitted to the emergency department for interpretation and intervention. Which results would req | ST elevation |
| The nurse administers morphine as ordered to a patient experiencing chest pain of 9 on a scale of 10 that is unrelieved by three doses of nitroglycerin. Which assessment finding would be most concerning? | Distended neck veins |
| Which components are part of an invasive hemodynamic monitoring system? | Arterial catheter Transducer Pressure infusion system Monitor |
| Which nursing student statement regarding hemodynamic monitoring requires further education? | “Pulmonary artery catheters are the least invasive hemodynamic monitoring approach.” |
| Which nursing statement is an accurate explanation of preload? | “Preload is the amount that the heart stretches.” |
| When assisting a provider in the placement of a central venous catheter, into what position will the nurse place the patient? | Trendelenburg |
| Which patient assessment data require immediate nursing intervention? | A continuous wedge pressure waveform This finding requires immediate nursing intervention because a continuous wedge pressure waveform indicates that the pulmonary artery may be occluded. |
| What is the most likely cause of a low-pressure alarm within an arterial hemodynamic monitoring system? | Disconnection Hemorrhage can occur with a disconnection in the system; this is the most likely cause of a low-pressure alarm. |
| The source of a patient’s dysrhythmias and heart failure (HF) has been attributed to a malfunction in the heart’s ability to contract. When providing an explanation to the patient, which statement by the nurse best explains this type of HF? | “This is systolic heart failure.” |
| A 61-year-old patient with high cholesterol, diabetes, hypertension, and a family history of lung cancer is attending an education class on healthy lifestyle changes and asks the nurse educator: “If I remain compliant with my choles | “That is important, but you also need to control your blood sugar.” |
| Which patients must the nurse monitor for the development of heart failure (HF)? | A patient with anemia A patient with hypothyroidism A patient with a pulmonary embolism |
| A patient with dilated cardiomyopathy is diagnosed with heart failure (HF) and is presenting with systemic complications. Which statement best explains the cause of these complications? | “The sympathetic nervous system may be activated in response to heart failure.” |
| Which statement by the nurse best explains dyspnea as one of the clinical presentations of acute decompensated heart failure (ADHF)? | "Left ventricle failure causes breathing problems."Left ventricle (LV) failure results in engorgement of the pulmonary vascular system, which can result in dyspnea. |
| The nurse is examining a patient with heart failure (HF). The patient asks, “The provider says I have poor cardiac output. Which of my symptoms are caused by this?” The nurse would explain to the patient that which symptoms are a direct re | Fatigue Chest pain Depression |
| A patient calls the clinic complaining of an inability to sleep and waking up in a panic. The nurse suspects heart failure (HF). Which additional questions should the nurse ask to support this decision? | “Are you experiencing any trouble breathing?” “Have you experienced a nonproductive cough?” When you breathe, are you able to breathe deeply? |
| A patient with acute decompensated heart failure (ADHF) is admitted to the unit. About which orders should the nurse ask for clarification? | -Regular diet -Normal saline IV bolus -Vital signs every 4 hours |
| A patient with heart failure (HF) is experiencing dyspnea and reports not being able to perform their tasks of daily living due to the symptoms. What nondrug recommendation by the nurse is appropriate for the care of this patient? | "Take some time off work and rest." |
| A patient with heart failure (HF) is admitted to the hospital, put on bed rest, administered oxygen, and treated with an angiotensin-converting enzyme (ACE) inhibitor. Additionally, the patient is educated about healthy lifestyle choic | -"It is much easier to breathe." -"I am able to walk to and from the restroom much easier now." -"I have been taking this medication as ordered since it was prescribed." |
| A patient with heart failure (HF) is being discharged with new medications and oxygen therapy. The patient reports still feeling weak and expresses concern about using the oxygen tank. About which collaborative care option should the n | Home health referral |
| Which laboratory result will the nurse assess prior to administering spironolactone to a patient with chronic heart failure (HF)? | Potassium |
| A patient with acute decompensated heart failure (ADHF) is admitted to the cardiac care unit and is prescribed furosemide 40 mg intravenous push STAT, which is available from the pharmacy as 10 mg/mL. | 4 mL |
| A patient with acute decompensated heart failure (ADHF) is receiving digoxin intravenously. Which assessment finding would the nurse expect if the medication is having a therapeutic effect? | Increased myocardial contractility |
| A nurse and patient are developing a comprehensive care plan for treating the patient’s heart failure (HF). The patient asks why it is necessary to monitor sodium intake. Which response by the nurse is best? | Sodium allows the body to retain more fluid. |
| Which foods should a patient taking a loop diuretic for management of chronic heart failure (HF) be encouraged to eat? | -bananas -dried fruit -orange juice |
| The nurse is preparing teaching for a patient hospitalized with severe heart failure (HF) with secondary renal insufficiency. Which will be important to include in the teaching plan regarding diet prior to discharge? | Limit fluid intake to 2 L per 24 hours. |
| A nurse is educating a patient about heart failure (HF). Which statement made by the patient demonstrates understanding of the disease? | “I may feel tired because of my decreased cardiac output.” |
| A patient is admitted to the cardiac unit with shortness of breath, fatigue, and dizziness due to heart failure (HF). The patient states, “I just need to be able to walk without being so tired so I can go back to work.” | Talk to the patient about diet and exercise changes Ask the patient about what their ideal day would look like Ask the patient's family to participate in a patient teaching session |
| A patient with heart failure (HF) is admitted to the hospital and the nurse is providing patient teaching. Which statement by the patient indicates that further education is necessary? | "We are retiring and moving to Phoenix, Arizona, to adopt a quiet, stress-free lifestyle." |
| A male patient’s acute decompensated heart failure has resolved and he is being transferred from the ICU to a medical floor. He is anxious that the nursing care won’t be as attentive on that floor and that his condition will w | “The nurses will assess your vital signs and breathing pattern every 2 to 4 hours.” |
| A patient on the medical unit with acute decompensated heart failure (ADHF) is becoming increasingly more dyspneic without relief from previous interventions. What is the most appropriate action by the nurse in the care of this patient? | Call the provider |
| A patient is being seen in the clinic after being discharged with acute decompensated heart failure 2 weeks ago. The patient states, “I can’t do what I used to do, so I just spend a lot of time by myself.” Which is the best response by the nurse? | “Do you feel sad or down?” |
| A patient with end-stage heart disease asks, “After I meet the physical criteria for a heart transplantation, will I then be selected as a candidate?” Which response from the nurse is the most appropriate? | “You will also need to demonstrate you have adequate support.” |
| The nurse is reviewing potential candidates for heart transplantation. Which patients have contraindications for undergoing heart transplantation? | a. A homeless patient with allergies c. An obese patient with a BMI of 42 kg/m2 e. A patient with uncontrolled diabetes mellitus (DM) |
| A patient with end-stage heart disease is inquiring about heart transplantation options. The patient has COPD and a history of bladder cancer. Which statement should the nurse make to gather more infor | “Have you been admitted to the hospital for your COPD?” |
| A nurse is teaching a class on transplantation protocols. Which scenarios should be used to explain retransplantation? | a. A heart transplantation recipient needs the donor heart replaced. d. A patient is receiving a second heart transplantation from another donor. |
| The nurse is assisting the transplantation team during heart transplantation surgery. Which finding would require immediate follow-up by the nurse? | Radial pulse is non-palpable of +1. |
| The nurse is caring for a patient undergoing a heart transplantation. Which statement by the patient would require immediate follow-up? | d. "I won't have to worry about infections once I stop taking these medications." |
| The nurse is caring for a patient returning for an evaluation 6 months after heart transplantation surgery. The patient reports compliance with the medication regimen. Which assessment finding would require immediate follow-up? | Enlarged lymph node |
| The nurse is caring for a patient who is next on the transplantation list and who is waiting at the hospital for a donor heart to become available. Which statement by the patient would be most concerning? | "I can stay at home since I only live about a 1-hour drive from the hospital." |
| The nurse is working on a heart transplantation team. After being notified that a donor heart has become available, how would the transplantation team proceed? Order the steps for a heart transplantation procedure. | Remove the donor's heart Attach cardiopulmonary bypass to recipient Remove the recipient's heart Implant the donor's heart into recipient |
| Which statement describes the role of the Purkinje fibers in the pathway of the normal cardiac cycle? | Conduction of impulses through the ventricles leading to ventricular muscle contraction |
| Which action results in the atria after firing of the sinoatrial (SA) node? | Contraction of the atria to move blood to the ventricles |
| Which physiologic process is responsible for repolarization of heart cells? | Movement of sodium out of the cell |
| The nurse is reviewing a patient’s ECG reading and notices artifact on the monitor. Which action should the nurse perform to identify the cause of the artifact? | Determine the condition of the conductive gel on the electrodes |
| A nursing student is discussing a rhythm strip with the faculty member. Which rationale explains a shortened QT interval? | A faster heart rate |
| Using the R-R interval formula, the heart rate in the strip shown measures beats per minute | 80 |
| The nurse obtains an ECG for a 53-year-old patient with chest pain. The nurse palpates a heart rate of 106. Which alteration in waveform should the nurse expect to see on the ECG tracing? | Short QT interval |
| The nurse is caring for a patient with chest tightness and shortness of breath. The ECG tracing reports sinus tachycardia. Which additional finding should be expected? | Heart rate is 114 beats per minute |
| The nurse is providing patient education regarding bradycardia and its management. Further teaching is required if the patient makes which statement? | “My heart rate may be slower if I discontinue my calcium channel blockers.” |
| A patient calls the health care provider’s office reporting dizziness and fatigue. The patient started taking a beta blocker one week ago. The patient’s radial pulse is 52 bpm. Which instruction should the nurse give the patient? | "Consult your health care provider about stopping the beta blocker." |
| A patient reports dizziness and dyspnea. On assessment, the patient’s heart rate is 130 bpm. An ECG indicates normal P-wave, PR interval, and QRS complex. The patient reports that the symptoms began shortly after an injury causing acute back pain. What | Administer pain medications as prescribed. |
| A patient presents with sinus tachycardia with a heart rate of 122. The patient also has pneumonia. Which assessment question should the nurse ask the patient first to identify the cause of the patient’s symptoms? | "Have you taken any over-the-counter medications since you became sick?" |
| A patient presents during a normal well-visit with a heart rate of 58 beats per minute. The patient is an avid runner and is currently training for a triathlon. All other vital signs are normal. Whi | "I do not need to take any medications. My heart rate is expected because of my triathlon training." |
| Match the type of atrial dysrhythmia with its ECG characteristics. | Atrial rate of 300 bpm and ventricular rate of 150 bpm A Flutter Irregular rhythm and indiscernible P wave PAC QRS complex appears normal, but PR interval is indiscernible A fib |
| Why might a patient with atrial fibrillation be prescribed warfarin? | To decrease stroke risk |
| The nurse is working with a patient who has a atrial rate of 500 beats per minute (bpm). The nurse expects which characteristic ECG abnormality? | fibrillatory P wave |
| A patient reports feeling palpitations and chest heaviness after several days of gastroenteritis. The nurse notes a serum potassium level of 3.1, and the patient’s ECG shows no visible P waves, a normal QRS complex, and an immeasurable PR in | Calcium channel blocker |
| A patient with a history of hypertension presents with chest tightness and discomfort, dyspnea, and dizziness. The ECG is shown. Which process is the primary goal when treating this patient? | Increase AV block The patient is experiencing atrial flutter. The primary goal in treatment of atrial flutter is to slow the ventricular response by increasing AV block. |
| A patient with a history of chronic obstructive pulmonary disease (COPD) presents with chest pain and signs of atrial flutter. The patient’s vital signs are blood pressure 65/40, temperature 99.0 °F, heart rate 135 beats per minute, and respiratory rat | Administer warfarin according to orders Prepare the patient for an electrical cardioversion Administer a dose of IV narcotic pain medication |
| A 59-year-old patient arrives at the emergency department and with chest discomfort for the past 24 hours. An ECG reading shows a heart rate of 58, a regular rhythm with a normal QRS complex, and an inverted P wave on the fourth beat. W | The impulse is coming from the AV node, because the SA node is either not functioning or the impulse is blocked. |
| A 65-year-old patient with a 20-year history of CAD receives an ECG that shows a variable PR interval with no relationship between the P wave and the QRS complex. The nurse notes which type of heart block? | Third-degree heart block In third-degree heart block, the PR interval is variable, and there is no relationship between the P wave and the QRS complex. |
| The nurse examines the patient’s ECG and notices that there is a junctional rhythm. Which area of the heart is controlling the heart rate? | AV node |
| A patient with a history of HF reports chest pain. An ECG shows a prolonged PR interval that remains constant on conducted beats. The QRS complex is 0.14 seconds. Which action does the nurse take for management of this patient? | Prepare the patient for insertion of a temporary pacemaker |
| A patient has the ECG strip shown here as most current in the chart and has been experiencing very low blood pressures. The nurse should expect to collaborate with the care team on which procedure for this patient? | Temporary pacemaker placement |
| A patient has blood pressure 88/56 and heart rate 33 bpm with the ECG strip shown here. After the appropriate care team members are notified, which intervention should be performed first? | Administer dopamine |
| A patient with a history of third-degree heart block comes to the ED with severe chest pain and bradycardia. Which serious medical complication does the nursing team need to prepare for in caring for this patient? | Shock |
| For which reasons may the nurse confuse ventricular fibrillation for asystole? | Patients are unresponsive in ventricular fibrillation and asystole. Patients are usually apneic with ventricular fibrillation and asystole. The irregularly shaped waves of ventricular fibrillation look similar to occasional P waves in asystole. |
| Why should the patient with a history of myocardial infarction (MI) be monitored closely when premature ventricular contractions (PVCs) are seen on the electrocardiogram (ECG) strip? | PVCs may develop into ventricular tachycardia. |
| Why is the heart rate undetermined for a patient in ventricular fibrillation (VF)? | P-to-P intervals and QRS complexes are not measurable. |
| A patient in ventricular fibrillation (VF) is receiving CPR. What is the priority intervention? | Prepare to defibrillate. |
| The nurse assesses the electrocardiogram (ECG) strip shown here and notes the patient has a palpable peripheral pulse. Which action should the nurse take first? | Apply 100% oxygen via non-rebreather. |
| Which action is priority for the nurse caring for a patient with this electrocardiogram (ECG) tracing? | Palpate the patient’s carotid pulse. |
| A nurse is assessing a patient who has been diagnosed with hypertrophic cardiomyopathy (CMP). Which is the primary defect in cardiac function? | Thickened ventricular walls |
| A nurse is caring for a patient with dilated cardiomyopathy. Which common symptoms might the nurse encounter? | Fatigue Anorexia Dyspnea at rest Blood clot formation Paroxysmal nocturnal dyspnea |
| A patient with myocardial fibrosis reports fatigue, exercise intolerance, and dyspnea. The nurse notes 3+ lower extremity edema and jugular venous distention. Which type of cardiomyopathy commonly presents with these symptoms? | Restrictive cardiomyopathy |
| The nurse is caring for a patient with cardiomyopathy related to chronic sarcoidosis. The patient reports fatigue and shortness of breath. The nurse notes dry mucous membranes and poor skin turgor. Which nursing action is the priority? | Administer intravenous (IV) normal saline solution at a rate of 75 mL/hr. |
| A community health nurse presents information on management of hypertrophic cardiomyopathy to a group. The nurse should include which information in the presentation? | Amiodarone Echocardiogram β-Adrenergic blockers Atrioventricular pacing Percutaneous transluminal septal myocardial ablation |
| The nurse is caring for a patient diagnosed with alcohol-related cardiomyopathy. The patient reports crushing chest pain, shortness of breath, and fatigue. Which provider order would the nurse implement first? | Administer nitroglycerin sublingually. |
| Which assessment finding would best describe a patient with pericarditis? | Pain worse with deep inspiration, relieved by sitting in upright position |
| The nurse is caring for a patient who suffered a myocardial infarction early Saturday morning. When would the nurse expect the patient to experience symptoms of acute pericarditis? | Monday afternoon |
| The nurse is caring for a patient with a history of strep throat. The patient asks the nurse how a strep infection can lead to heart disease. Which is the best response by the nurse? | "Strep bacteria cause scarring and deformity to the layers of the heart." |
| A patient diagnosed with endocarditis on IV antibiotic therapy for 2 days presents with a fever. Which interventions would the nurse implement to treat the associated fever? | Administer aspirin Give acetaminophen |
| A patient diagnosed with pericarditis is not responding to medical therapy with NSAIDs. Which therapy would the nurse anticipate? | Corticosteroids |
| A patient with myocarditis presents with BP 195/94, crackles heard in lungs bilaterally, and edema of the lower extremities. Which strategy should the nurse implement first? | Administering diuretics |
| A patient diagnosed with pericarditis reports being extremely anxious and upset. Which responses can the nurse use given the patient’s emotional anxiety? | "I see you seem anxious and upset." "I will stay with you until your family arrives." |
| A nurse is caring for a patient who has blood cultures indicate infective endocarditis (IE ). Which nursing intervention should the nurse perform first? | Start intravenous (IV) antibiotic therapy |
| A multidisciplinary team is working with a patient who has endocarditis. The multidisciplinary team is providing teaching about complication prevention. Which statement by the patient indicates the need for further teaching? | "I plan to visit my 8-year-old grandson in Michigan this January." |
| Which subjective symptoms would the nurse recognize as being associated with chronic mitral valve regurgitation? | Dyspnea Chest pain Generalized fatigue |
| The nurse is caring for a patient with diagnosed aortic regurgitation. Which patient history finding would the nurse note as a possible cause? | Untreated strep pharyngitis |
| The nurse is caring for a patient who reports frequent angina, dyspnea on exertion, and peripheral edema. The nurse notes a systolic murmur on auscultation and crackles in the lungs. Which diagnostic test would the nurse anticipate? | Transesophageal echocardiogram |
| A nurse is assessing an older adult patient with mitral valve stenosis and a history of unstable angina. Which therapy would the nurse anticipate as a conservative therapy for this patient? | Percutaneous transluminal balloon valvuloplasty |
| The nurse is caring for a 72-year-old patient with a history of alcohol-related cirrhosis and who has been diagnosed with aortic stenosis. Previous drug therapy and valve repair were unsuccessful. Which procedure would the nurse anticipate? | Biologic valve replacement |
| A 32-year-old female patient with valvular heart disease has been treated with metoprolol and digoxin for several years. After undergoing transcutaneous valvuloplasty, the patient continues to experience dyspnea and pulmonary edema. Which treatment | Biologic valve replacement |
| The nurse is caring for a patient with mitral stenosis and who experiences nausea, muscle weakness, and chest heaviness. The nurse notes an apical pulse of 58 bpm. Which action is most important for the nurse to take? | Hold the PO digoxin |
| The nurse is caring for a patient after valve replacement surgery. The patient reports chest pain 6/10 and mild dyspnea. The nurse notes oxygen saturation 92% on 2L via NC, HR 68 bpm, BP 86/42 mmHg, and respirations 16 per minute. Which action | Give IV normal saline bolus |
| The nurse is performing a follow-up assessment for a patient after open surgical valvuloplasty. Which assessment findings would be concerning? | Dry cough Orthopnea Peripheral edema Saturated surgical dressing |
| The patient is admitted to the hospital with a chondrosarcoma. In which part of the body would the nurse expect to find this bone tumor? | Cartilage in the arm, leg, or pelvis |
| A nurse is evaluating a list of patients and identifies which patient as having the highest risk for bone cancer? | A 10-year-old boy with a hard mass on his shoulder |
| A patient asks the nurse about the development of a benign bone tumor. Which information should the nurse provide? | Benign tumors will not spread to other body parts. |
| A nurse is caring for a patient who has a 20-pack-year (one pack a day for 20 years) history of smoking. The patient reports shortness of breath and back pain when taking a deep breath. Initial testing has revealed a mass in the spine. Whic | Computed tomography (CT)-guided biopsy |
| The nurse is caring for a patient whose malignant tumor is not operable. Which procedures would the nurse expect the health care provider to recommend for this patient? | Radiation Chemotherapy Pain management Regular screenings |
| A health care provider has ordered a computed tomography (CT) scan of the leg of a patient with suspected osteochondroma. Which question should the nurse ask the patient first before the procedure? | “Do you have allergies to shrimp or other seafood?” |
| The nurse is caring for a patient who is being considered for a limb salvage of the right lower extremity. Which elements are priority for the nurse to assess before the procedure? | Pulse Movement Circulation |
| A child is admitted to the hospital for surgical resection of an osteosarcoma and will require chemotherapy. The child is fearful of the hospital admission. Which action should the nurse take? | Encourage the family to stay with the child. |
| The nurse is coordinating the care of a patient with a metastatic tumor to the hip who is scheduled for chemotherapy infusion in the morning. Prioritize the steps in the care of this patient. | Take patient's blood pressure Clean and access port Begin chemotherapy infusion Provide patient education |
| A child is admitted with a diagnosis of osteochondroma and is crying inconsolably because of pain. Which action should the nurse take first? | Assess the location and severity of pain |
| A nurse is caring for an older adult patient and suspects the patient may have a herniated disc. Which signs/symptoms are indicative of a herniated disc? | Pain Numbness Leg weakness |
| The nurse is providing orientation to a new graduate who is caring for a patient with a herniated disc. The graduate asks, “What is a disc herniation?” Which is the appropriate response by the nurse? | "The nucleus pulposus has bulged out through an opening in the annulus fibrosus." |
| The nurse would most likely expect to see vertebral discs break down in which region of the spine? | Lumbar |
| When preparing to care for a patient with intervertebral disc disease, what should the nurse plan to tell the patient to do in order to maintain proper body mechanics? | Sit upright while seated at a desk |
| The nurse is providing health education to a patient who is scheduled for disc replacement. Which information should the nurse include in the teaching plan about the procedure? | Disc replacement will reduce, but not eliminate, pain. |
| A patient being prepared for a laminectomy tells the nurse, “I only took six days off from work to recover. Will that be enough time?” Which response by the nurse is appropriate? | "It may take a few weeks to fully recover from the surgery. Six days may not be enough time." |
| The nurse is caring for a patient who has undergone cervical spine surgery. Which findings should prompt the nurse to implement immediate life-saving measures? | Dyspnea Nasal flaring |
| The nurse is assessing the dressing of patient after completion of lumbar spine surgery and notes yellow-tinged drainage. After further examination, the nurse documents that the patient has a headache with a rating of seven on a pain scale of one to | Notify the health care provider immediately |
| The medical-surgical charge nurse receives a patient in the postanesthetic care unit after successful completion of spinal fusion. The patient is still sedated and is assigned to a staff nurse. The unit charge nurse intervenes when the s | Diazepam, 2.5 mg, by mouth |
| Which findings would the nurse expect to note when assessing the lower extremities of a patient with low back pain? | Tense muscle tone Hyperactive reflexes Guarding of the area |
| The nurse is reviewing the intake forms for a patient with acute low back pain. Which finding is most likely related to the cause of the pain? | Twisted funny while playing volleyball |
| The student nurse asks the instructor, “Why are nurses at such great risk for developing low back pain?” Which response by the instructor is correct? | “Nurses often perform a lot of heavy lifting and bending.” |
| A patient with low back pain is beginning chiropractic therapy. The provider prescribes cyclobenzaprine, 10 mg every eight hours as needed, for muscle stiffness or pain. The medication is available in 5-mg tablets. The patient asks the nurse, “How ma | 6/day |
| A patient with chronic low back pain is complaining of difficulty sleeping. Which medication would be an appropriate treatment for the patient’s complaint? | Duloxetine |
| The nurse is caring for a patient with acute low back pain. Which topics would be included when teaching the patient about management techniques? | Safe use of oral opioid medications Attending physical therapy sessions Information on complementary therapy |
| The nurse is developing a plan of care for a patient admitted to the hospital with chronic back pain. Which interventions should be included in the plan? | Assess patient's pain level offer hot and cold compress provide pillows while the patient is sleeping |
| The patient with acute low back pain tells that nurse that a new job requires stocking shelves with boxes of equipment. Which information about proper body mechanics should the nurse give the patient? | “Do not lift boxes past the elbows.” |
| The nurse walks into the room of a patient with low back pain. The patient is packing belongings and preparing for discharge. The nurse notes the patient is leaning forward with the legs straight, picking up shoes off the floor. Which action should th | Correct the patient by reminding him or her to bend the knees when leaning forward. |
| Place the clinical progression of rheumatoid arthritis in order of occurrence in the disease process. | 1. Synovial inflammation and lymphocytes increase. 2. Articular cartilage destruction causes growth of vascular granulation tissue. 3. Inflamed pannus destroys bone and bone edges. 4. Joint deformities occur. |
| A patient diagnosed with osteoarthritis (OA) would have which manifestations? | Joint pain and stiffness Grating sensation with movement Nodes at the proximal interphalangeal joints |
| Which manifestations would indicate that a patient is in the late stage of rheumatoid arthritis (RA)? | Gel phenomenon Peripheral neuropathy |
| An image of the hand of a patient with osteoarthritis (OA) is provided. Which label (A, B, C, or D) in the image points to a Bouchard’s node? | Label C points to a Bouchard’s node, which is located at the proximal interphalangeal joint. This deformity is commonly found in patients with OA. |
| A patient with osteoarthritis (OA) reports doing a lot of walking and is now experiencing increased knee pain. Which actions should the nurse recommend to this patient? | Use a whirlpool on a daily basis, alternate application of ice and heat to the knee, take frequent rest periods with the knee elevated |
| Which teaching should the nurse provide to a patient with rheumatoid arthritis (RA) who has just started taking azathioprine? | Avoid people who are sick |
| A female patient with rheumatoid arthritis (RA) states that she has been treated for years with different medications but is frustrated that she still hasn't achieved relief. Which responses from the nurse are appropriate? | "Maybe you should try yoga or Tai Chi." "You should ask your health care provider about joint replacement." "You can speak to your health care provider about transcutaneous electrical nerve stimulation (TENS)." |
| Which questions should the nurse ask a patient with osteoarthritis (OA) to determine comfort? | "When your pain occurs, how long does it last?" "How would you rate your pain on a scale of 0 to 10?" "What medications do you take to alleviate your arthritic pain?" "Does the arthritis prevent you from doing activities you enjoy?" |
| A patient with osteoarthritis (OA) who is working as an office assistant asks the nurse what can be done to perform the job while managing their symptoms of OA. Which responses from the nurse would benefit the patient? | "Use a computer instead of writing by hand." "Use a voice or video recorder and transcribe when necessary." "Sit up straight and make sure you have your office chair in a comfortable position." |
| Which patient statement indicates correct understanding about the management of rheumatoid arthritis (RA)? | "I have started taking a yoga class weekly." |
| The nurse is caring for a patient with a new diagnosis of systemic lupus erythematosus (SLE). The patient denies a family history of this disease. Which information in the patient's history identifies a likely cause of the onset of symptoms? | Use of prescribed procainamide |
| A patient with systemic lupus erythematosus (SLE) asks the nurse what happens during an exacerbation of SLE. Which response from the nurse is appropriate? | “Your body is making antibodies that attack the proteins in your DNA.” |
| Which finding in a patient with systemic lupus erythematosus (SLE) would indicate possible kidney damage? | Protein in the urine Proteinuria is a common finding in patients with SLE and indicates possible kidney damage. |
| A patient diagnosed with systemic lupus erythematosus (SLE) is taking methotrexate. Which instruction should the nurse provide to the patient? | "You need to take a folic acid supplement." Methotrexate depletes folic acid. Therefore supplementation is required. |
| A nurse is caring for a patient with systemic lupus erythematosus (SLE) who reports having mild pain and swelling in the hands. Which medication would the nurse anticipate the health care provider to prescribe? | Celecoxib Celecoxib is a nonsteroidal antiinflammatory drug that is prescribed to treat arthritic pain related to SLE. |
| A female patient comes to the emergency department with a butterfly rash and tests positive for antinuclear antibodies. Which additional tests would the nurse anticipate to be ordered by the health care provider to confirm a diagnosis of systemic lu | Urinalysis can confirm whether there is protein or any other indicator of kidney disease in the urine, which is a positive indicator of SLE. Serum complement test- A serum complement test will confirm serositis, which is a positive indicator of SLE. |
| The nurse has established a goal of verbalizing an understanding of activities to prevent exacerbations with a female patient with systemic lupus erythematosus (SLE). Which statement made by the patient would support this goal? | “I plan to do meditation daily and take time for myself.” |
| A nurse is caring for a patient diagnosed with systemic lupus erythematosus (SLE) who has been on a long-term regimen of methylprednisolone. Which precautions should the nurse take to prevent complications of the treatment? | Assess the patient's mental status- Monitor the patient's blood glucose levels- |
| A patient with systemic lupus erythematosus (SLE) mentions to the nurse that she is trying to conceive. Which questions should the nurse ask to address potential pregnancy complications? | Are you currently taking methotrexate? Have you had an antiphospholipid antibody test? Have you and your partner started pregnancy counseling. |
| The nurse caring for a patient diagnosed with squamous cell carcinoma knows that which patient findings are risk factors for the development of skin cancer? | Inability to tan Celtic ancestry Infrequent use of sunscreen |
| The nurse is caring for a patient diagnosed with melanoma. The nurse is aware that which areas are common sites for this cancer? | Back Scalp Trunk Lower legs |
| The nurse teaches the patient the ABCDEs of skin self-examination. Which statement by the patient demonstrates that teaching has been effective? | “I should report any moles that are more than ¼ inch in diameter to my health care provider.” |
| A patient has returned to the dermatology office to review results of a repeat skin biopsy. The provider is unable to determine the results of the biopsies on microscopic examination. Whic | The patient is taking ibuprofen. |
| A patient underwent an excisional biopsy 3 days ago. The patient calls the clinic and reports a scant amount of bleeding and “a little discomfort.” What is the best action by the nurse? | Inform the patient that slight bleeding and discomfort are expected after an excisional biopsy |
| A patient has been diagnosed with malignant melanoma. The health care provider orders adjuvant therapy after surgery for this patient. Which medication does the nurse anticipate administering to the patient? | Cisplatin |
| Which nursing interventions are most appropriate for a patient receiving psoralen therapy? | Provide skin moisturizers Administer ondansetron as ordered Recommend the use of an emollient |
| A patient with skin cancer undergoing radiation therapy tells the nurse that she has become self-conscious about changes in her skin. What recommendations can the nurse make to the patient so that she can avoid further skin damage? | Wear a scarf on your head to protect your scalp.” “Find some activities that will help you reduce stress levels.” “Use gentle, nonirritating soaps when you wash your hands and bathe.” |
| The nurse is most concerned about the administration of psoralen to which patient? | A patient with alcohol-related liver disease |
| A patient shares with the nurse a concern about a skin tag on the inner thigh. The patient is becoming worried that the skin tag is cancerous. How should the nurse respond? | “Most skin tags are harmless and won’t require any treatment, unless they are irritating and you want them removed.” |
| Which practice should the nurse emphasize as being the single most important means of preventing secondary skin infections? | Handwashing |
| A young adult female was recently prescribed oral birth control pills. The patient states that since she started taking the pills, her skin has become “problematic.” The nurse understands that which skin disorder is a common side effect | Acne vulgaris |
| The nurse is concerned that a patient is experiencing complications of long-term oral corticosteroid use. Which finding indicates a possible complication? | Blood glucose of 168 mg/dL Long-term oral corticosteroid use can increase blood glucose levels. The elevated blood glucose of 168 mg/dL is a possible complication of oral corticosteroid use. |
| Phototherapy for a patient with atopic dermatitis has been effective when the nurse notes which assessment finding? | Itching has resolved. |
| A patient presents to the clinic with welts on both arms and itchy, watery eyes and is diagnosed with allergic contact dermatitis. The patient receives a prescription but a few days later complains of feeling drowsy with the medication and requests a di | Cetirizine Loratadine Fexofenadine Hydrocortisone |
| The nurse is taking care of a patient with impetigo. Which methods are appropriate for management of the skin lesions and discomfort? | Topical antibiotics Warm saline soaks |
| Which statement indicates the nurse is performing proper patient teaching during discharge of a patient who has recently undergone a dermatologic procedure? | “Check the site for any type of pus or drainage and call the health care provider immediately if you see it.” |
| The nurse notes that a patient appears very withdrawn and depressed. When the nurse approaches the patient, the patient states, “I’ll never look the same again with all these skin issues and treatments. I don’t even want to leave this | “Cosmetics can be helpful in situations like this. Would you like to discuss this more?” |
| A patient with gastritis asks the nurse about the bodily process behind the diagnosis. The nurse explains that which physiologic event is responsible for the patient’s symptoms? | Disruption of the stomach mucosa |
| The nurse would identify which parts of the medical and social history as placing a patient at increased risk for development of gastritis? | habitual use of NSAIDs previous Helicobacter pylori infection consumption of spicy food |
| A patient is taking a corticosteroid. Which action of the corticosteroid places the patient at risk for development of gastritis? | Inhibition of prostaglandin synthesis |
| A patient complains of burning pain across the midepigastric area 3 hours after eating dinner. The pain is relieved by antacids. Which diagnosis would the nurse suspect based on these symptoms? | Duodenal ulcer |
| A patient with a suspected Helicobacter pylori infection and complications is being prepared for an endoscopy. The patient asks why the procedure is being done. How can the nurse respond? | "This study will confirm the infection is present." "We need to take a sample of the stomach lining." "The procedure allows us to determine whether there is any internal damage." |
| A patient is suspected of having a gastric ulcer. The patient cannot have an endoscopy because a non-life-threatening cardiac arrhythmia is present. Which diagnostic test should the patient undergo? | Barium contrast study |
| Which action by the nurse demonstrates the first step in providing care to a patient with nausea and a new diagnosis of gastritis? | Confirming medical history accuracy |
| The care team is treating a post-operative patient who has recently undergone a gastrectomy. The patient is showing signs of anxiety and lab results reveal excessive amounts of insulin. The nurse understands that these symptoms may be indicative | Postprandial hypoglycemia |
| A nurse is treating a patient with acute gastritis. Which information should the nurse provide to help the patient prevent future exacerbations? | eat only bland food join a smoking cessation support group graze on food throughout the day |
| If a patient presents with acute exacerbation of peptic ulcer disease, which collaborative interventions need to be taken? | Give patient NPO orders Nasogastric (NG) suction Administration of intravenous Ringer’s lactate solution Intravenous administration of a proton pump inhibitor |
| A patient presents 2 weeks after removal of a large portion of the stomach and pyloric sphincter. The patient is complaining of abdominal cramping, dizziness, and a sensation of fullness. On assessment, the patient’s heart rate is 125 bpm | Dumping syndrome |
| A patient with peptic ulcer disease (PUD) presents with severe upper abdominal pain that spreads to the back. The patient reports that they cannot find any way to relieve the pain. On assessment, the abdomen is rigid and the patient has tachycardia | Omentum graft |
| A patient presents with vomiting and nausea. On assessment, the nurse notes epigastric tenderness. Endoscopic studies confirm infection with H. pylori. The patient is prescribed triple-drug therapy. What patient education topic is a priority to ens | importance of completing antibiotics |
| A patient is diagnosed with acute gastritis associated with a diet heavy in spicy foods and is currently experiencing severe nausea. Which treatment is appropriate for this patient? | nasogastric tube |
| A patient with gastritis asks the nurse how to prevent future exacerbations. The patient is a heavy smoker and drinker, takes an aspirin daily for heart problems, and eats a relatively simple and bland diet. The patient walks 1 mile every night f | Stop drinking Cease smoking |
| Why would the nurse discuss vitamin B12 deficiency with a patient who has undergone a gastrectomy? | The parietal cells of the stomach produce hydrochloric acid (HCl), which is an essential factor for vitamin B12 absorption. |
| A patient presents with a history of hypertension and has just started treatment for peptic ulcer disease (PUD). What important medication topic needs to be discussed in the management of this patient? | anatacids need to be used with caution |
| Early detection of peptic ulcer disease (PUD) needs to be the priority in a nursing management plan for patients with symptoms of chronic gastritis. Which action is the nurse’s next priority? | assess for anemia and guaiac-positive stools |
| Which assessment findings would the nurse expect to find for a patient with diagnosed Crohn’s disease that has developed complications outside the gastrointestinal (GI) tract? | -joint pain -red patches on legs -eye redness & itching |
| The laboratory results of a patient with ulcerative colitis (UC) indicate anemia. The nurse determines that which factor is most likely responsible for this laboratory finding? | Blood loss |
| Which risk factors will the nurse include when planning to teach a group of teenagers about inflammatory bowel disease (IBD)? | -stress -high-fat diet -cigarette smoking |
| Which is the rationale for the nurse to instruct the patient with inflammatory bowel disease (IBD) who is receiving antibiotics to eat yogurt daily? | To prevent the occurrence of Clostridium difficile infection |
| The nurse is caring for a patient with suspected inflammatory bowel disease (IBD). Which assessment finding warrants immediate health care provider notification? | Potassium level 2.6 mEq/L |
| A patient with Crohn’s disease reports passing stool when urinating. Which surgical treatment would the nurse anticipate? | Fistula repair |
| A patient with inflammatory bowel disease (IBD) presents with a weight loss of 10 pounds in the past month. The patient reports poor appetite and frequent diarrhea. Which lab value is most concerning? | Albumin level 2.2 g/dL |
| The nurse is caring for a patient with inflammatory bowel disease (IBD) who reports frequent diarrhea, nausea, poor appetite, and a weight lost 10 pounds in the last 2 weeks. The nurse notes pallor, poor skin turgor, mild peripheral edema, and dry m | -Calculate intake and output -Infuse intravenous (IV) albumin -Administer intravenous (IV) fluid |
| The nurse is caring for a patient with inflammatory bowel disease (IBD) who reports frequent diarrhea and vomiting and a pain level of 8/10. The nurse notes poor skin turgor, dry mucus membranes, and abdominal tenderness to palpation. Whi | Administer pain medication |
| The nurse is caring for a patient with inflammatory bowel disease (IBD) who reports palpitations and weakness. The nurse notes a thready, irregular heartbeat. Which lab result would the nurse review first? | Serum potassium level |
| A patient presents to the emergency department and reports frequent small light-colored stools and fever. On examination, the nurse notes jaundice of the skin and sclera and a white blood cell (WBC) count of 15,000 /μL. An ultrasound examination | Prepare the patient for an endoscopic retrograde cholangiopancreatography (ERCP) |
| During assessment of a patient for suspected cholelithiasis with biliary colic, a positive response to which question posed by the nurse would support the diagnosis? | “Do you have pain after consuming a high-fat meal?” |
| A 55-year-old obese African American woman has been admitted for a cholecystectomy to treat cholecystitis. The patient asks the nurse what factors might have predisposed her to cholecystitis. Which factors would the nurse mention? | Being female Being overweight Over the age of 40 |
| What would be the best collaborative care intervention for a patient admitted for treatment of an acute episode of cholecystitis who reports dry skin and severe itching? | Administer cholestyramine |
| female patient scheduled for a laparoscopic cholecystectomy tells the nurse she is frightened because she has heard from others that the abdominal distention and pain after the surgery are severe. Which car | “We will help you walk in the hall to decrease distention.” |
| A patient who is not a candidate for surgery is admitted to the hospital with acute cholecystitis and severe nausea and vomiting. Which collaborative care intervention will help maintain fluid and electrolyte balances? | Lactated Ringers |
| The nurse is preparing to administer an injection to a patient being treated for obstructive choledocholithiasis. The patient has been having severe episodes of nausea and vomiting and has a temperature of 100.2° F and bleeding gums. | Gentle pressure 23g needle |
| A patient who has undergone laparoscopic cholecystectomy asks the nurse about getting up and taking a shower. Which response by the nurse would be appropriate? | “It is best to do a bedside bath because you want to keep the dressing dry” |
| Which statement made by the student nurse when caring for a patient who has had an open cholecystectomy requires correction? | resume normal activities in 2 weeks |
| Which nursing interventions will be most effective to alleviate nausea and vomiting by decreasing gallbladder stimulation in a patient with cholecystitis? | NG NPO |
| The caregiver of a patient with suspected pancreatitis asks the nurse why both serum amylase and serum lipase tests are being done if they are both enzymes related to the pancreas. Which explanation by the nurse accurately explains why both are being don | “The serum amylase can be high due to other causes, so we also run the serum lipase.” |
| Which statement best describes the pathophysiology of acute pancreatitis? | Inflammation activates enzymes, resulting in autodigestion within the pancreas. |
| The nurse is evaluating a patient in the emergency department for suspected pancreatitis. Which factors, if present in the patient history or current assessment, would increase the risk for pancreatitis? | History of gallstones Presence of biliary sludge African American ethnicity 30 pack year history of smoking |
| Following teaching, the nurse is evaluating the patient’s understanding of the need for both an endoscopic retrograde cholangiopancreatography (ERCP), plus endoscopic sphincterotomy, and a laparoscopic cholecystectomy. Which statement by | "A laparoscopic cholecystectomy visualizes the colon, and the ERCP removes the gallstones." |
| The nurse is reviewing the current prescriptions for a patient with pancreatitis. Following a recent assessment, the patient was found to have decreased bowel sounds and abdominal distention. Which current medication order would the nurse | Dicyclomine |
| The nurse is awaiting orders for a patient being admitted from the emergency room for conservative therapy for pancreatitis. The nurse would include which goals in the plan of care for the newly admitted patient? | Patient will remain free of infection. Patient will verbalize understanding of NPO status. Fluid and electrolyte levels are within normal limits. Patient will rate pain at less than 2 in a 24-hour period. |
| Which assessment finding would demonstrate the efficacy of pancreatic enzyme replacement? | No evidence of steatorrhea |
| A patient presents to the primary care provider one week after hospital discharge for an episode of acute pancreatitis. The patient reports a fever that has lasted for the last few days and stools have had a “really foul smell” and “appear greasy.” A | Further pancreatic destruction |
| The nurse is caring for a patient with diagnosed acute pancreatitis and notes a recent elevation in temperature, a respiratory rate of 28 breaths/minute, and a few crackles in the bases of the lungs. Which actions by the nurse would best address the pati | Obtain O2 saturation level Elevate the bed to semi-Fowler's position Instruct patient on incentive spirometry use |
| What should the nurse include in a focused assessment when caring for a patient admitted to the hospital with a diagnosis of cirrhosis? | Assess the lower extremities. Assess skin integrity and color. Measure abdominal circumference. |
| Which pathologic change would cue the nurse to assess for excess fluid volume in a patient with cirrhosis? | Hyperaldosteronism |
| An elevation in which laboratory value can lead to increased confusion and lethargy in the patient with cirrhosis? | Ammonia |
| The nurse is preparing to administer a unit of fresh frozen plasma to a patient with cirrhosis admitted for a transjugular intrahepatic portosystemic shunt (TIPS) procedure. The patient’s caregiver asks why the patient will be receiving fresh fro | “Fresh frozen plasma provides the patient with the necessary clotting factors that the liver is not producing.” |
| The nurse is preparing to administer spironolactone to a patient with cirrhosis. Which finding would lead the nurse to withhold the dose of spironolactone? | Hyperkalemia |
| A nurse is reviewing medications with a patient who is being discharged after having an esophageal variceal bleed. The patient questions why propranolol is being prescribed when blood pressure is low. What is the nurse’s best response? | “Propranolol will help decrease the portal pressure to prevent further bleeding from existing varices.” |
| A nurse is caring for a patient with a long-standing history of cirrhosis. Which findings on admission would cue the nurse to ask for further information from the patient or caregiver? | Uses acetaminophen for pain Lethargic and oriented to person Consumes 1 to 2 alcoholic beverages per week |
| The nurse is providing discharge teaching to a male patient with cirrhosis about managing his care at home. Which statement made by the patient indicates a need for further teaching? | "I will weigh myself every other day." |
| The nurse is reviewing the admission history and physical examination findings for a patient with a history of cirrhosis who is being admitted for the third time in 6 months. Which laboratory finding would indicate the patient has not foll | Blood alcohol level 160 mg/dL |
| Which interventions should the nurse include in the plan of care for a patient with cirrhosis who is experiencing excess fluid volume? | Obtain a daily weight. Auscultate lung sounds. Elevate lower extremities. |